June 2017 - Watching yourself on the treadmill

barlow__barlow__ Member

The correct answer on this question doesn't make sense to me. The main conclusion of the argument is very narrow, and the answer goes outside the scope of the argument in order to weaken it. Can someone please explain this to me? Maybe in a private message? I'm trying not to break any rules by revealing too much here.

Comments

  • inactiveinactive Alum Member
    12637 karma

    What section and question# is this? Hard to help when we don't know where to look.

  • OlamHafuchOlamHafuch Alum Member
    2326 karma

    I actually challenged this question, and just today received a response from LSAC. I can PM their response to anyone who is interested in seeing it.

  • OlamHafuchOlamHafuch Alum Member
    2326 karma

    I should add that my offer is only for those who took the June LSAT, and therefore already have legal access to the text of the question. I'll be happy to PM everyone else after PT 81 is released to the public.

Sign In or Register to comment.